Very tough CR -- M LOST

This topic has expert replies
Senior | Next Rank: 100 Posts
Posts: 68
Joined: Tue Jul 19, 2011 9:47 am
Thanked: 6 times

Very tough CR -- M LOST

by need720+ » Fri Aug 12, 2011 10:58 am
Which of the following most logically completes the argument?

A certain cultivated herb is one of a group of closely related plants that thrive in soil with high concentrations of metals that are toxic to most other plants. Agronomists studying the herb have discovered that it produces large amounts of histidine, an amino acid that,in test-tube solutions, renders these metals chemically inert. Possibly, therefore, the herb's high histidine production is what allows it to grow in metal-rich soils, a hypothesis that would gain support if ______.

A. histidine is found in all parts of the plant-roots, stem, leaves, and flowers
B. the herb's high level of histidine production is found to be associated with an unusually low level of production of other amino acids
C. others of the closely related group of plants are also found to produce histidine in large quantities
D. cultivation of the herb in soil with high concentrations of the metals will, over an extended period, make the soil suitable for plants to which the metals are toxic
E. the concentration of histidine in the growing herb declines as the plant approaches maturity

Newbie | Next Rank: 10 Posts
Posts: 2
Joined: Tue May 10, 2011 4:39 am

by singh151151 » Fri Aug 12, 2011 11:57 am
Answer should be C:

The stem mentions that this herb(say X) is part of a group of plants and all the plants in that group can thrive in soil with high concentration of metals that are toxic to most other plants,

If say, other plants in the group are not found to contain large amounts of histidine,then the reason this group of plants(including X) can thrive in such soils is something other than histadine

Legendary Member
Posts: 784
Joined: Sun Apr 03, 2011 3:51 am
Thanked: 114 times
Followed by:12 members

by patanjali.purpose » Fri Aug 12, 2011 3:47 pm
need720+ wrote:Which of the following most logically completes the argument?

A certain cultivated herb is one of a group of closely related plants that thrive in soil with high concentrations of metals that are toxic to most other plants. Agronomists studying the herb have discovered that it produces large amounts of histidine, an amino acid that,in test-tube solutions, renders these metals chemically inert. Possibly, therefore, the herb's high histidine production is what allows it to grow in metal-rich soils, a hypothesis that would gain support if ______.

A. histidine is found in all parts of the plant-roots, stem, leaves, and flowers
B. the herb's high level of histidine production is found to be associated with an unusually low level of production of other amino acids
C. others of the closely related group of plants are also found to produce histidine in large quantities
D. cultivation of the herb in soil with high concentrations of the metals will, over an extended period, make the soil suitable for plants to which the metals are toxic
E. the concentration of histidine in the growing herb declines as the plant approaches maturity
IMO D

if in long run the cultivation of herb makes the metals chemically inert , then it means histidine is effective

C - weaken the argument

Master | Next Rank: 500 Posts
Posts: 125
Joined: Fri Aug 12, 2011 6:11 pm
Thanked: 8 times

by crick » Sun Aug 14, 2011 5:15 am
I will go with E here.

Conclusion : High Histidine production allows plant to grow in metal-rich soils. => Only purpose of histidine is to allow the plant to grow in the metal-rich soil.

E states that as the plant continues its growth, the histidine production declines => In effect, indicating that indeed histidine just allows the plant to start growing in metal-rich soil.

Crick

Master | Next Rank: 500 Posts
Posts: 218
Joined: Sat Jul 24, 2010 2:43 pm
Thanked: 5 times

by cyrwr1 » Sun Aug 14, 2011 7:47 pm
D is my response.

It is saying H allows plants to grow in metal/toxic areas.

If this is true, then D can be used as a premise.

All feedback appreciated.

User avatar
Master | Next Rank: 500 Posts
Posts: 344
Joined: Mon Sep 14, 2009 5:40 am
Thanked: 28 times
Followed by:3 members
GMAT Score:700

by sunnyjohn » Sun Aug 14, 2011 8:12 pm
Answer will definitely be [spoiler]"C".[/spoiler]

If my answer is correct, I will write down my explanation. Anyway - very nice Question.

Junior | Next Rank: 30 Posts
Posts: 29
Joined: Tue May 31, 2011 1:44 am
Thanked: 2 times

by gmat062011 » Sun Aug 14, 2011 9:51 pm
IMO D.

User avatar
Newbie | Next Rank: 10 Posts
Posts: 1
Joined: Thu Jun 30, 2011 2:22 pm

by Jthap » Sun Aug 14, 2011 10:46 pm
Definitely E ..
This proves that Histidin is essential for the growth of plants. The level of histidine will drop once the plant approaches its maturity; hence, less histidine will be required for the plant to grow.

User avatar
Legendary Member
Posts: 1079
Joined: Mon Dec 13, 2010 1:44 am
Thanked: 118 times
Followed by:33 members
GMAT Score:710

by bblast » Mon Aug 15, 2011 12:07 am
This is a poorly constructed question from the GMAC banned resource - scoretop sets. Stay away !!
Cheers !!

Quant 47-Striving for 50
Verbal 34-Striving for 40

My gmat journey :
https://www.beatthegmat.com/710-bblast-s ... 90735.html
My take on the GMAT RC :
https://www.beatthegmat.com/ways-to-bbla ... 90808.html
How to prepare before your MBA:
https://www.youtube.com/watch?v=upz46D7 ... TWBZF14TKW_

Legendary Member
Posts: 581
Joined: Sun Apr 03, 2011 7:53 am
Thanked: 52 times
Followed by:5 members

by killer1387 » Sun Aug 21, 2011 8:31 pm
D is what i feel
OA please...!!!

User avatar
Legendary Member
Posts: 540
Joined: Sat Dec 20, 2008 7:24 pm
Thanked: 37 times
Followed by:6 members

by navami » Mon Aug 22, 2011 6:15 am
According to me it gt to be D
This time no looking back!!!
Navami

Legendary Member
Posts: 581
Joined: Sun Apr 03, 2011 7:53 am
Thanked: 52 times
Followed by:5 members

by killer1387 » Tue Aug 23, 2011 5:23 am
IMO its E coz it supports the conclusion.

Legendary Member
Posts: 2789
Joined: Tue Jul 26, 2011 12:19 am
Location: Chennai, India
Thanked: 206 times
Followed by:43 members
GMAT Score:640

by GmatKiss » Tue Aug 23, 2011 6:03 am
IMO:C !

Legendary Member
Posts: 2330
Joined: Fri Jan 15, 2010 5:14 am
Thanked: 56 times
Followed by:26 members

by mundasingh123 » Tue Aug 23, 2011 8:36 am
bblast wrote:This is a poorly constructed question from the GMAC banned resource - scoretop sets. Stay away !!
Why did GMAC ban scoretop sets ? I dont think the reason warrants our staying away from this source .
I Seek Explanations Not Answers

Legendary Member
Posts: 2330
Joined: Fri Jan 15, 2010 5:14 am
Thanked: 56 times
Followed by:26 members

by mundasingh123 » Tue Aug 23, 2011 8:54 am
However i hate this question i can try to justify all the choices C , D and E .
why did someone who liked this CR like this CR ?
I Seek Explanations Not Answers